0% found this document useful (0 votes)
131 views11 pages

Problems: MN) MN

The document contains problems related to limit theorems from a statistics textbook. Problem 4 asks how the sample size n recommended by the Chebyshev inequality would change under two scenarios: if the population fraction being estimated was reduced to half its original value, and if the probability tolerance in the Chebyshev inequality was reduced to half its original value. Both changes would require increasing the sample size n. The remaining problems ask about determining whether certain sequences of random variables converge in probability and identifying their limits under different scenarios.

Uploaded by

Nouran Y
Copyright
© © All Rights Reserved
We take content rights seriously. If you suspect this is your content, claim it here.
Available Formats
Download as PDF, TXT or read online on Scribd
0% found this document useful (0 votes)
131 views11 pages

Problems: MN) MN

The document contains problems related to limit theorems from a statistics textbook. Problem 4 asks how the sample size n recommended by the Chebyshev inequality would change under two scenarios: if the population fraction being estimated was reduced to half its original value, and if the probability tolerance in the Chebyshev inequality was reduced to half its original value. Both changes would require increasing the sample size n. The remaining problems ask about determining whether certain sequences of random variables converge in probability and identifying their limits under different scenarios.

Uploaded by

Nouran Y
Copyright
© © All Rights Reserved
We take content rights seriously. If you suspect this is your content, claim it here.
Available Formats
Download as PDF, TXT or read online on Scribd
You are on page 1/ 11

284 Limit Theorems Chap.

PROBLEMS

SECTION 5.1. Some Useful Inequalities


Problem 1. A statistician wants to estimate the mean height h (in meters) of a
population, based on n independent samples Xl , . . . , X chosen uniformly from the
n,

entire population. He uses the sample mean Mn = (Xl + . . . + X ) /n as the estimate


n

of h, and a rough guess of 1 .0 meters for the standard deviation of the samples Xt •
(a) How large should n be so that the standard deviation of Mn is at most 1 cen­
timeter?
(b) How large should n be so that Chebyshev's inequality guarantees that the esti­
mate is within 5 centimeters from h, with probability at least 0.99?
(c) The statistician realizes that all persons in the population have heights between
1 .4 and 2.0 meters, and revises the standard deviation figure that he uses based
on the bound of Example 5.3. How should the values of n obtained in parts (a)
and (b) be revised?

Problem 2. * The Chernoff bound. The Chernoff bound is a powerful tool that
relies on the transform associated with a random variable, and provides bounds on the
probabilities of certain tail events.
(a) Show that the inequality

P ( X � a) ::; e-s a M(s)

holds for every a and every s � 0, where M(s) = E[e S X ] is the transform associ­
ated with the random variable X, assumed to be finite in a small open interval
containing s = O.
(b) Show that the inequality

P ( X ::; a) ::; e - s a M(s)

holds for every a and every s ::; O.


(c) Show that the inequality

holds for every a, where

¢(a) = max (sa - In 1\.1 ( s)) .


s 2: 0

(d) Show that if a > E[X] , then ¢(a) > O.


Problems 285

(e) Apply the result of part (c) to obtain a bound for P(X � a ) , for the case where
X is a standard normal random variable and a > 0.
(f) Let Xl , X2 , . . . be independent random variables with the same distribution as
X. Show that for any a > E[X], we have

so that the probability that the sample mean exceeds the mean by a certain
amount decreases exponentially with n.
Solution. (a) Given some a and s � 0, consider the random variable Ya defined by
if X < a,
if X � a.

It is seen that the relation

always holds and therefore,

On the other hand,

from which we obtain

(b) The argument is similar to the one for part (a) . We define Ya by
{ e sa , if X � a,
Ya =
0, if X > a.

Since s � 0, the relation

always holds and therefore,

On the other hand,

from which we obtain


P(X � a ) � e-s a M(s).
286 Limit Theorems Chap. 5

(c) Since the inequality from part (a) is valid for every s � 0, we obtain
P(X � a) � min
s�o
(e - sa j\,f (s))

= mIn. e - ( sa - In M( s ))
s�o

- e max s -> O ( sa - In M ( s ) )
_ -

- e -q)( a ) .
_

(d) For s = 0, we have


sa - In M(s) = O - ln l = 0,
where we have used the generic property M(O) = 1 of transforms. Furthermore,
d d
ds ( sa - ln M(s)) l s=O = a - M(s)
- I -M(s)
ds
-_

I s =O = a - 1 · E[X] > o.
.

Since the function sa - In M (s) is zero and has a positive derivative at s = 0, it must be
positive when s is positive and small. It follows that the maximum ¢(a) of the function
sa - In M (s) over all s � 0 is also positive.
(e) For a standard normal random variable X , we have M(s) = e s 2 /2 • Therefore,
sa - In M(s) = sa - s 2 /2. To maximize this expression over all s � 0, we form the
derivative, which is a - s, and set it to zero, resulting in s = a. Thus, ¢(a) = a2 /2,
which leads to the bound

Note: In the case where a � E [XJ , the maximizing value of s turns out to be s = 0,
resulting in ¢( a) = 0 and in the uninteresting bound
P(X � a) � 1 .
(f) Let Y = Xl + . . . + Xn Using the result of part (c), we have
.

P (* i?' a) = � n
P(Y � na) :o e -> y ( n) ,

where
¢y (na ) = max
s�O
(nsa - In My (s) ) ,
and

=
is the transform associated with Y. We have In My (s) n ln M(s) , from which we
obtain
¢y ( )na . maxO ( sa - In M (s)) n¢( )
= n = a ,
s�
Problems 281

and
p (;; t ) Xi ::: a :; , - n .( a )

Note that when a > E [XJ , part (d) asserts that ¢ (a) > 0, so the probability of interest
decreases exponentially with n.
Problem 3. * Jensen inequality. A twice differentiable real-valued function J or a
single variable is called convex if its second derivative ( d2 J Idx2 )( x ) is nonnegative for
all x in its domain of definition.
(a) Show that the functions J(x ) eOx, J( x ) - In x, and J( x ) = X 4 are all convex.
= =

(b) Show that if J is twice differentiable and convex, then the first order Taylor
approximation of J is an underestimate of the function. that is,
d
J(a) + ( x - a) dxJ (a) � J( x ).

for every a and x.


(c) Show that if J has the property in part (b), and if X is a random variable, then
J (E [X I) � E [J(X) ] .

Solution. (a) We have


d2
dx 2
e ax = a2 e ax > 0 , d2
dx 2 (
- In x ) = 2"
1
x
> 0,
d2 X4
dx 2
= 4 . 3 . x2 � o.

(b) Since the second derivative of J is nonnegative, its first derivative must be nonde­
creasing. Using the fundamental theorem of calculus, we obtain
J( x ) = J(a) + J.x : (t) dt � J(a) + J.x : (a) dt = J(a) + ( x - a) �� (a).

(c) Since the inequality from part (b) is assumed valid for every possible value x of the
random variable X , we obtain
J(a) + ( X - a) �� (a) � J( X ) .

We now choose a = E[X ] and take expectations, to obtain

J (E [X I) + (E[X ] - E[XI) �� (E[Xl) � E [J(X) ] ,


or
J (E[X l) � E [ J(X) ] .
288 Limit Theorems Chap. 5

SECTION 5.2. The Weak Law of Large Numbers

Problem 4. In order to estimate f. the true fraction of smokers in a large population,


Alvin selects n people at random. His estimator Aln is obtained by dividing Sn. the
number of smokers in his sample, by n, i.e., Mn Sn ln . Alvin chooses the sample
=

size n to be the smallest possible number for which the Chebyshev inequality yields a
guarantee that

where € and 6 are some prespecified tolerances. Determine how the value of n recom­
mended by the Chebyshev inequality changes in the following cases.
(a) The value of f is reduced to half its original value.
(b) The probability 6 is reduced to half its original value.

SECTION 5.3. Convergence in Probability

Problem 5. Let Xl . X2, be independent random variables that are uniformly


distributed over 1-1. 1]. Show that the seqllence Y1 , Y2 converges in probability to
• • •

some limit, and identify the limit, for each of the following cases;
• • • •

(a) Yn = Xn ln .
(b) Yn = (Xn) n .
(c) Yn = Xl ' X2 X• . •
n.

(d) Yn = max{Xl " " , Xn } .


Problem 6. * Consider two sequences of random variables X l . X2 , . . . and Y1 , Y2 , . . . ,
which converge in probability to some constants. Let be another constant . Show that
e

cXn. Xn + Yn . max{O, Xn} . IXn l . and XnYn all converge in probability to corresponding
limits.
Solution. Let x and y be the limits of Xn and Yn , respectively. Fix some € > 0 and a
constant If = 0, then cXn equals zero for all n, and convergence trivially holds. If
e. e

e =f:. 0, we observe that p (lcXn - exl 2: f) = p ( IXn - xl 2: f/lel ) , which converges to


zero. thus establishing convergence in probability of cXn.
We will now show that P(IX" + y" - x - yl 2: f) converges to zero, for any € > O.
To bound this probability. we note that for IXn + Yn - X - yl to be as large as f, we
need either IXn - xl or lY.l - xl (or both) to be at least €/2. Therefore, in terms of
events. we have

This implies that

and
nlim lim p (IXn - xl 2: €/2) + nlim
-+� p (IXn + Y,, - x - yl 2: €) � n-+oo -+oc
p (IYn - y l � €/2) = 0,
Problems 289

where the last equality follows since Xn and Yn converge, in probability, to x and y ,
respectively.
By a similar argument, it is seen that the event { I max{O, Xn } - max{O, x}1 2: } E

is contained in the event { I Xn - x l 2: E } . Since limn-Xl p ( IXn - xl ;:::: E ) = 0, this


implies that
limoo P ( l max{O. Xn } - max{O, x}I 2: E ) = 0.
n-
Hence max{O, Xn } converges to max{O, x} in probability.
We have IXn l = max{O, Xn }+max{O, -Xn}. Since max{O, Xn} and max{O, -Xn }
converge, as shown earlier, it follows that their sum, IXn I, converges to max {O, x} +
max{O. -x} = Ixl in probability.
Finally, we have
P ( IXnYn - xyl 2: E ) = P ( I (Xn - X)(Yn - y) + xYn + yX" - 2xyl ;:::: E )
::; P ( I (Xn - X ) (YrI - y ) 1 2: E / 2) + p (lxYn + yXn - 2xy l 2: E/2 ) .
Since xYn and yXn both converge to xy in probability. the last probability in the above
expression converges to 0. It will thus suffice to show that

To bound this probability. we note that for I(Xn - X)(Yn - y ) 1 to be as large as E/2, we
need either IXn - xl or IYn - xl (or both) to be at least v;J2. The rest of the proof
is similar to the earlier proof that Xn + Yn converges in probability.
Problem 7. * A sequence Xn of random variables is said to converge to a number c

in the mean square, if


lim [ ( Xn - c) 2 ] = 0.
n -oo E

(a) Show that convergence in the mean square implies convergence in probability.
(b) Give an example that shows that convergence in probability does not imply con­
vergence in the mean square.
Solution. (a) Suppose that Xn converges to c in the mean square. Using the Markov
inequality, we have

Taking the limit as n - x. we obtain


n-limX p ( IXn - cl 2: E) = 0.
which establishes convergence in probability.
(b) In Example 5.8, we have convergence in probability to ° but E [Y;] = n3 , which
diverges to infinity.
290 Limit Theorems Chap. 5

SECTION 5.4. The Central Limit Theorem


Problem 8. Before starting to play the roulette in a casino, you want to look for
biases that you can exploit. You therefore watch 100 rounds that result in a number
between 1 and 36. and count the number of rounds for which the result is odd. If the
count exceeds 55, you decide that the roulette is not fair. Assuming that the roulette is
fair, find an approximation for the probability that you will make the wrong decision.
Problem 9. During each day. the probability that your computer's operating system
crashes at least once is 5%, independent of every other day. You are interested in the
probability of at least 45 crash-free days out of the next 50 days.
( a) Find the probability of interest by using the normal approximation to the bino­
mial.
( b ) Repeat part ( a) , this time nsing the Poisson approximation to the binomial.
Problem 10. A factory produces Xn gadgets on day n. where the Xn are independent
and identically distributed random variables, with mean 5 and variance 9.
( a ) Find an approximation to the probability that the total number of gadgets pro­
duced in 100 days is less than 440.
( b ) Find ( approximately ) the largest value of n such that
P (X I + . . . + Xn 2: 200 + 5n) ::; 0.05.

(c) Let N be the first day on which the total number of gadgets produced exceeds
1000 . Calculate an approximation to the probability that N 2: 220.
Problem 1 1 . Let X YI , X2 . Y2 •
I. be independent random variables, uniformly
distributed in the unit interval [0. l ] , and let
• • •

W=
( Xl + . . . + X 1 6) - ( YI
16
+ . . . + Y16 )
.
Find a numerical approximation to the quantity
p(I"" - E[ltV1 1 < 0.001) .

Problem 12. * Proof of the central limit theorem. Let X X . be a sequence


of independent identically distributed zero-mean random variables with common vari­
(j2 ,
I,
2, . .

ance and associated transform Alx (s) . We assume that Mx (s) is finite when
-d < s < d, where d is some positive number. Let

( a) Show that the transform associated with Zn satisfies


Afz,, (s) = ( ( (jfo))
Afx n
Problems 291

( b) Suppose that the transform Mx (s) has a second order Taylor series expansion
around s = 0, of the form

where o ( S2 ) is a function that satisfies lims -+o o ( S2) / S2 = O. Find b, and in a, c


terms of a2•
( c ) Combine the results of parts ( a) and ( b ) to show that the transform lvlzn (s)
converges to the transform associated with a standard normal random variable,
that is,
nlim
-+oo MZn (s) = es / 2 , for all s.
2

No t e: The central limit theorem follows from the result of part ( c ) , together with the
fact ( whose proof lies beyond the scope of this text ) that if the transforms MZn (s)
converge to the transform Mz (s) of a random variable Z whose CDF is continuous,
then the CDFs FZn converge to the CDF of Z. In our case, this implies that the CDF
of Zn converges to the CDF of a standard normal.
Solution. ( a) We have, using the independence of the Xi ,

MZn (s) = E [e S zn ]

=E [exp { ��t,x'}l
n

i=l

(b ) Using the moment generating properties of the transform, we have


d
a = Mx (O) = 1 , b= d
s
Mx (s) 1 s=o = E[X] = 0,
and

(c) We combine the results of parts ( a) and ( b) . We have


MZn (s) = Mx
( ( )) ( +
a fo
s
n
= a
bs cs 2
a fo + a 2 n +0
( ))
S2
a2 n
n
'

and using the formulas for a, b, and c from part ( b ) , it follows that
292 Limit Theorems Chap. 5

We now take the limit as n - 00 , and use the identity


lim ( + � )
n oo 1 n
n
= eC,
.....

to obtain

SECTION 5.5. The Strong Law of Large Numbers


Problem 13. * Consider two sequences of random variables Xl , X2 , . . . and YI , Y2 ,
Suppose that Xn converges to a and Yn converges to b, with probability 1. Show that
. • • •

Xn + Yn converges to a+b, with probability 1. Also, assuming that the random variables
Yn cannot be equal to zero, show that Xn / Yn converges to a l b, with probability 1.
Solution. Let A ( respectively, B) be the event that the sequence of values of the
random variables Xn ( respectively, Yn ) does not converge to a (respectively, b) . Let C
be the event that the sequence of values of Xn + Yn does not converge to a + b and
notice that C C A U B .
Since Xn and Yn converge to a and b, respectively, with probability 1. we have
P (A ) = 0 and P(B) O. Hence,
=

P ( C ) :s; P(A U B) :s; P (A ) + P(B ) = O.


Therefore. P ( CC ) 1, or equivalently, Xn + Yn converges to a + b with probability 1.
For the convergence of Xn / Yn , the argument is similar.
=

Problem 14. * Let Xl . X2 , . . be a sequence of independent identically distributed


random variables. Let YI , Y2 , be another sequence of independent identically dis­
.

tributed random variables. We assume that the Xl and � have finite mean, and that
• • •

YI + . . . + Yn cannot be equal to zero. Does the sequence


Xl + . . . + Xn
YI + . . . + Yn
Zn = -::-:----....­
..,.-

converge with probability 1, and if so, what is the limit?


Solution. We have
(Xl + . . . + Xn )/n
Zn = (YI + . . . + Yn )/n .
By the strong law of large numbers, the numerator and denominator converge with
probability 1 to E[X] and E[Y] . respectively. It follows that Zn converges to E[XJjE[Y] ,
with probability 1 (cf. the preceding problem ) .
Problem 15. * Suppose that a sequence YI , Y2 , of random variables converges to
a real number with probability 1. Show that the sequence also converges to in
• • •

c, c

probability.
Solution. Let C be the event that the sequence of values of the random variables Yn
converges to By assumption, we have P( C) = 1. Fix some € > 0, and let Ak be
c.

the event that I Yn l < € for every n � k. If the sequence of values of the random
c

variables Yn converges to then there must exist some k such that for every n � k,
-

c,
Problems 293

this sequence of values is within less than E from c. Therefore, every element of C
belongs to Ak for some k, or
U Ak.
00

Cc
k= 1
Note also that the sequence of events Ak is monotonically increasing, in the sense
that Ak C Ak+ l for all k. Finally, note that the event A k is a subset of the event
{ I Yk - cl < E } . Therefore,
lim p ( I Yk - cl < E ) � lim P(Ak) = P(U�= I Ak ) � P(C) = 1,
k -oo k -oc
where the first equality uses the continuity property of probabilities (Problem 13 in
Chapter 1). It follows that

lim p ( I Yk - cl � E ) = 0,
k-oc
which establishes convergence in probability.
Problem 16. '" Consider a sequence Yn of nonnegative random variables and suppose
that

Show that Yn converges to 0, with probability 1 .


Note: This result provides a commonly used method for establishing convergence with
probability 1 . To evaluate the expectation of 2::'= 1 Yn , one typically uses the formula

E [t. ] �
Yn = E[Yn ) .

The fact that the expectation and the infinite summation can be interchanged, for
the case of nonnegative random variables, is known as the monotone convergence
theorem. a fundamental result of probability theory, whose proof lies beyond the scope
of this text.
Solution. We note that the infinite sum 2::'= 1 Yn must be finite, with probability
1 . Indeed. if it had a positive probability of being infinite, then its expectation would
also be infinite. But if the sum of the values of the random variables Yn is finite, the
sequence of these values must converge to zero . Since the probability of this event is
equal to 1 , it follows that the sequence Yn converges to zero, with probability 1 .
Problem 17. '" Consider a sequence of Bernoulli random variables Xn , and let pn =
P(Xn = 1) be the probability of success in the nth trial. Assuming that 2::'= 1 pn < 00 ,
show that the number of successes is finite, with probability 1 . [Compare with Problem
48(b) in Chapter 1 .]
Solution. Using the monotone convergence theorem (see above note) , we have
294 Limit Theorems Chap. 5

This implies that


00

L Xn < 00 ,
n =l
with probability 1 . We then note that the event { 2:::'1 Xn < oo } is the same as the
event that there is a finite number of successes.
Problem 18. * The strong law of large numbers. Let Xl , X2, . . . be a sequence
of independent identically distributed random variables and assume that E[XtJ < 00 .
Prove the strong law of large numbers.
Solution. We note that the assumption E[XtJ < 00 implies that the expected value of
the Xi is finite. Indeed, using the inequality Ixl � 1 + X4 , we have
E [IXi l] � 1 + E[XtJ < 00 .

Let us assume first that E[XiJ = o. We will show that

E L [
X (Xl + . . . + Xn) 4 <
n4
] 00 .

n=l
We have
[ 4 ]
E (Xl + . n. 4. + Xn) = n14 � ���
L- L- L- L- E[Xi l Xi 2 X' 3 Xi4 J ·
' 1 = 1 '2 = 1 13 = 1 '4 = 1
Let us consider the various terms in this sum. If one of the indices is different from
all of the other indices, the corresponding term is equal to zero. For example, if i l is
different from i2, i3, or i4 , the assumption E[XiJ = 0 yields
E[Xi l Xi2 Xi3 Xi4 J = E[Xil JE[Xi2 Xi3 Xi4 J = o .
Therefore, the nonzero terms in the above sum are either of the form E[xtl (there are
n such terms) , or of the form E[Xl XJl, with i =f j . Let us count how many terms
there are of this form. Such terms are obtained in three different ways: by setting
il = i2 =f i3 = i4, or by setting i l = i3 =f i2 = i4, or by setting i l = i4 =f i2 = i3 . For
each one of these three ways, we have n choices for the first pair of indices, and n - 1
choices for the second pair. We conclude that there are 3n(n - 1 ) terms of this type.
Thus,
E [(Xl + . . . + Xn) 4 ] nE[Xt l + 3n(n - I)E[X� X�l .
=
2 2
Using the inequality xy � (x + y ) / 2, we obtain E[Xf xiJ � E[XtJ, and
E [(XI + . . . + Xn) 4 ] � (n + 3n(n - 1 ))E [Xtl � 3n2 E [Xt J .
It follows that

E [f; (X, + .�� + Xn)4 ] = f; �4 E [(XI + . . . + xn)4] f; :2 ElxtJ � < 00 .

2
where the last step uses the well known property 2:�=1 n - < 00. This implies that
(Xl + . . . +Xn) 4 /n4 converges to zero with probability 1 (cf. Problem 16), and therefore,
(Xl + . . . + Xn)/n also converges to zero with probability 1 , which is the strong law of
large numbers.
For the more general case where the mean of the random variables Xi is nonzero,
the preceding argument establishes that ( X l + . . . + Xn - nE [Xl.l )/n converges to zero,
which is the same as (X l + . . . + Xn)/n converging to E[Xl], with probability 1 .

You might also like

pFad - Phonifier reborn

Pfad - The Proxy pFad of © 2024 Garber Painting. All rights reserved.

Note: This service is not intended for secure transactions such as banking, social media, email, or purchasing. Use at your own risk. We assume no liability whatsoever for broken pages.


Alternative Proxies:

Alternative Proxy

pFad Proxy

pFad v3 Proxy

pFad v4 Proxy